What is the x-intercept of the line y=-2x+6? (3,0) -6,0) (0,3) (-3,3)

Answers

Answer 1

The given equation is expressed as

y = - 2x + 6

The x intercept of a line is the point at which y = 0

By applying this concept, it means that

0 = - 2x + 6

Adding 2x to both sides of the equation, it becomes

0 + 2x = - 2x + 2x + 6

2x = 6

Dividing both sides by 2, it becomes

2x/2 = 6/2

x = 3

Therefore, the correct option is (3, 0)


Related Questions

Mrs. algebra ordered some small and medium pizzas for her daughter‘s birthday party. The small pizzas cost $5.75 each and the medium pizzas cost $8.00 each. She bought three more medium pizzas than small pizzas and her total order came to $51.50 How many pizzas of each did Mrs. Algebra order? Write an equation and solve.

Answers

we have the following:

x is small pizzas

y is medium pizzas

[tex]\begin{gathered} 5.75\cdot x+8\cdot y=51.5 \\ y=x+3 \\ 5.75\cdot x+8\cdot(x+3)=51.5 \\ 5.75x+8x+24=51.5 \\ 13.75x=51.5-24 \\ x=\frac{27.5}{13.75} \\ x=2 \end{gathered}[/tex]

therefore, the answer is:

2 small pizzas and 5 (2+3) medium pizzas

At noon a private plane left Austin for Los Angeles, 2100 km away, flying at 500 km/h. One hour later a jet left Los Angeles for Austin at 700 km/h. At what time did they pass each other?

Answers

So if you take the L and do the w it should work

Which is an x-intercept of the continuous function in thetable?O (-1,0)O (0, -6)O (-6, 0)O (0, -1)

Answers

The x-intercept happens when:

[tex]f(x)=0[/tex]

Therefore, the x-intercepts for that functions are:

[tex]\begin{gathered} x=-1 \\ x=2 \\ x=3 \end{gathered}[/tex]

Answ

Figure RSTU has coordinates R = (3,4), S = (7.2), T = (5, 10), and U = (12,8). The figure is dilated from the origin by a scale factor r . Select the correct coordinates of R. A R' = (3,2) B R' = (1.5, 4) C R' = (3.5, 4.5) D R' = (1.5, 2) * Select the correct answer. 1 point Ο Α OB D

Answers

Answer

Option D is correct.

R' (1.5, 2)

Explanation

A dilation means the size is increased or decreased. If the scale factor is less than 1, then the size is decreased, but if the scale factor is more than 1, it means the figure is enlarged.

Dilating about the origin just multiplies the coordinates by the scale factor. So, dilating (x, y) about the origin by a scale factor k, gives new coordinates (kx, ky).

For this question, we need to dilate R (3, 4) by a scale factor, r = ½

R' = [½(3), ½(4)] = (1.5, 2)

Hope this Helps!!!

Using the image, identify the opposite rays (choose all that apply).

Answers

Solution

The answer is

We have a deck of 10 cards numbered from 1-10. Some are grey and some are white. The cards numbered are 1,2,3,5,6,8 and 9 are grey. The cards numbered 4,7, and 10 are white. A card is drawn at random. Let X be the event that the drawn card is grey, and let P(X) be the probability of X. Let not X be the event that the drawn card is not grey, and let P(not X) be the probability of not X.

Answers

Given:

The cards numbered are, 1,2,3,5,6,8, and 9 are grey.

The cards numbered 4,7 and 10 are white.

The total number of cards =10.

Let X be the event that the drawn card is grey.

P(X) be the probability of X.

Required:

We need to find P(X) and P(not X).

Explanation:

All possible outcomes = All cards.

[tex]n(S)=10[/tex]

Click boxes that are numbered 1,2,3,5,6,8, and 9 for event X.

The favourable outcomes = 1,2,3,5,6,8, and 9

[tex]n(X)=7[/tex]

Since X be the event that the drawn card is grey.

The probability of X is

[tex]P(X)=\frac{n(X)}{n(S)}=\frac{7}{10}[/tex]

Let not X be the event that the drawn card is not grey,

All possible outcomes = All cards.

[tex]n(S)=10[/tex]

Click boxes that are numbered 4,7, and 10 for event not X.

The favourable outcomes = 4,7, and 10

[tex]n(not\text{ }X)=3[/tex]

Since not X be the event that the drawn card is whic is not grey.

The probability of not X is

[tex]P(not\text{ }X)=\frac{n(not\text{ }X)}{n(S)}=\frac{3}{10}[/tex]

Consider the equation.

[tex]1-P(not\text{ X\rparen}[/tex][tex]Substitute\text{ }P(not\text{ }X)=\frac{3}{10}\text{ in the equation.}[/tex][tex]1-P(not\text{ X\rparen=1-}\frac{3}{10}[/tex][tex]1-P(not\text{ X\rparen=1}\times\frac{10}{10}\text{-}\frac{3}{10}=\frac{10-3}{10}=\frac{7}{10}[/tex]

[tex]1-P(not\text{ X\rparen is same as }P(X).[/tex]

Final answer:

[tex]1-P(not\text{ X\rparen is same as }P(X).[/tex]

hours worked. pay2. 12.504. 25.006. 37.508. 50.00write a function rule for the table

Answers

The Function is the equation of a line

Step1: we pick any two-point in the of the table and substitute them into the formula below

[tex]\frac{y-y_1}{x-x_1}=\frac{y_2-y_1_{}}{x_2-x_1}[/tex]

(y - y₁) / (x - x₁) = (y₂ - y₁) / (x₂ - x₁)

Let us pick the point (4,25) and (8,50)

where

[tex]x_1=[/tex]

Base on table above is the scenario a proportional relationship

Answers

Answer:

No

Explanations:

A relationship is called a proportional relationship if it has two variables that are realated by the same ration. In this case there will be a proportionality constant.

In this table:

Let Height be represented as H

Let Time be represented as T

For the relationship to be a proportional relationship, it must obey the relation:

[tex]\begin{gathered} H\propto\text{ T} \\ H\text{ = kT} \\ \text{Where k is the proportionality constant} \end{gathered}[/tex]

When T = 3, H = 15

Using H = kT

15 = 3k

k = 15 / 3

k = 5

When T = 6, H = 30

H = kT

30 = 6k

k = 30 / 6

k = 5

When T = 12, H = 45

H = kT

45 = 12k

k = 45 / 12

k = 3.75

Since the constant of proportionality is the the same for the three cases in the table, the scenario is not a proportional relationship

Problem: A school has a student to teacher ratio of25:5. If there are 155 teachers at the school, howmany students are there?Mike's AnswerCarlos's Answer25 .5 1551552555x = 3875x=77525x = 775x=31There are 31 students at the school.There are 775 students at the scheel.Who is correct? Mike or Carlos? Explain the error thatwas made.

Answers

In this case, we'll have to carry out several steps to find the solution.

Step 01:

Data:

ratio = 25:5 (students:teachers)

teachers = 155

students = ?

Step 02:

[tex]\begin{gathered} \text{students = 155 teachers }\cdot\text{ }\frac{25\text{ students }}{5\text{ teachers}} \\ \text{students = }775\text{ } \end{gathered}[/tex]

Carlos is correct.

[tex]\frac{25}{5}=\frac{x}{155}[/tex]

The answer is:

There are 775 students.

Carlos is correct.

Mike set the variables to find in the wrong way.

A 14 m long ladder is placed against a tree. The top of the ladder reaches a point
13 m up the tree.

How far away is the base of the ladder from the base of the tree?

Give your answer in metres (m) to 1 d.p.

Answers

Answer:

Approximately 5.2 meters

Step-by-step explanation:

This formation will make a right triangle. The ground to the point in the tree is one of the legs. The base of the tree to the base of ladder is another leg and the length of the ladder is the hypotenuse. In this case, we already have the hypotenuse and one of the legs, so we need to find the value of another leg.

We can do so by using the Pythagorean Theorem which is [tex]a^2+b^2=c^2\\[/tex].

a and b represent the values of the two legs, and c is the hypotenuse. Since we already have the hypotenuse, we can change this equation a bit to find the other leg.

Let's assign the missing value, b in the theorem.

The new equation will be [tex]b^2=c^2-a^2[/tex].

We can insert the values for c and a and solve for b.

The new equation will be [tex]b^2 = 14^2-13^2[/tex].

[tex]b^2=196-169[/tex]

[tex]b^2=27[/tex]

[tex]\sqrt{b^2} =\sqrt{27}[/tex]

The square root of [tex]b^2[/tex] cancels out.

The approximate square root of 27 is 5.19 which we can round to 5.2.

A water tank holds 276 gallons but is leaking at a rate of 3 gallons per week. A second water tank holds414 gallons but is leaking at a rate of 5 gallons per week. After how many weeks will the amount of waterin the two tanks be the same?The amount of water in the two tanks will be the same inweeks.

Answers

In order to solve the problem we will first create equations to represent the volume of water on the gallons through the weeks. The output of the functions will be the volume of each and the entry will be the number of weeks passed.

For the first one:

[tex]\text{vol(week) = 276 -3}\cdot week[/tex]

While on the second one:

[tex]\text{vol(week) = 414 -5}\cdot week[/tex]

In order to calculate the number of weeks it'll take until they have the same volume of water we need to find the "week" which would make them equal. So we will equate both expressions and solve for that variable.

[tex]\begin{gathered} 276\text{ - 3}\cdot week\text{ = 414 - 5}\cdot week \\ 5\cdot\text{week - 3}\cdot week\text{ = 414 - 276} \\ 2\cdot\text{week = }138 \\ \text{week = }\frac{138}{2}\text{ = }69 \end{gathered}[/tex]

It'll take 69 weeks for the tanks to have the same volume.

Write 6.5123 x 10^8 in standard

Answers

The standard form is a standard method of writing numbers such that we have it in the form:

[tex]a\times10^b[/tex]

where

[tex]0Therefore, 6.5123 x 10^8 in standard form is:[tex]6.5123\times10^8[/tex]

Name a postulate or theorem that can be used with the given information to prove that the lines are parallel<3 ~ <7

Answers

Postulates and Theorems of Parallel Lines

First, we need to know what type of angles are <3 and <7. Following the definition:

If two lines are crossed by another line, the angles in matching corners are called Corresponding Angles.

Angles 3 and 7 are corresponding angles and they are told to be congruent.

Now we apply the postulate that reads:

The Converse of the Corresponding Angles Postulate. If two lines are cut by a transversal so that corresponding angles are congruent, then the lines are parallel.

The postulate that can be used to prove that the lines are parallel is The Converse of the Corresponding Angles Postulate

For the point P (-12,22) and Q (-7, 27)​, find the distance​ d(P,Q) and the coordinates of the midpoint M of the segment PQ. What is the distance?

Answers

The distance​ d(P,Q) is equal to 7.1 units and the coordinates of the midpoint M of the segment PQ are (-9.5, 24.5).

How to determine the distance​ between points P and Q?

Mathematically, the distance between two (2) points that are located on a coordinate plane can be calculated by using this formula:

Distance = √[(x₂ - x₁)² + (y₂ - y₁)²]

Substituting the given parameters into the formula, we have;

Distance, d(P, Q) = √[(-7 + 12)² + (27 - 22)²]

Distance, d(P, Q) = √[5² + 5²]

Distance, d(P, Q) = √[25 + 25]

Distance, d(P, Q) = √50

Distance, d(P, Q) = 7.1 units.

Midpoint on x-coordinate is given by:

xm = (x₁ + x₂)/2

xm = (-7 - 12)/2

xm = -19/2

xm = -9.5

Midpoint on y-coordinate is given by:

ym = (y₁ + y₂)/2

ym = (27 + 22)/2

ym = 49/2

ym = 24.5

Therefore, the coordinates of the midpoint M are equal to (-9.5, 24.5).

Read more on distance here: https://brainly.com/question/7134030

#SPJ1

The perimeter of a rectangle is 48 centimeters. The relationship between the length, the width, and the perimeter of the rectangle can be described with the equation 2⋅length+2⋅width=48. Find the length, in centimeters, if the width is w centimeters

Answers

Using the relationship between the dimension of a rectangle and its perimeter, given its perimeter and width, the length is: 20.4 cm.

Recall:

Perimeter of a rectangle (P) = 2(L + W) (relationship between the width, length and perimeter)

Given:

Width (W) = 3.6 cm

Perimeter (P) = 48 cm

Length (L) = ?

Using the relationship between the dimension of a rectangle and its perimeter, the following equation would be derived:

48 = 2(L + 3.6)

Solve for the value of L

48 = 2L + 7.2

Subtract 7.2 from each side

48 - 7.2 = 2L

40.8 = 2L

Divide both sides by 2

20.4 = L

L = 20.4 cm

Therefore, using the relationship between the dimension of a rectangle and its perimeter, given its perimeter and width, the length is: 20.4 cm.

Learn more here:

brainly.com/question/18869010

Without needing to graph determined the number of solutions for this system

Answers

Given the system of equations:

[tex]\text{ x + y = 6}[/tex][tex]\text{ y = -x + 6}[/tex]

The two equations appear to be just the same, thus, we are only given one system of equations.

Therefore, the answer is letter B. It has infinite solutions because the two equations are just the same line.

solve the absolute value inequity lx-5l>_ 1

Answers

We are given the following the following inequality:

|x - 5| >= 1

When we have a inequality in the format:

|f(x)| >= a

There are two possible solutions.

Either f(x) <= -a or f(x) >= a

In this question:

|x - 5| >= 1

x - 5 <= -1

x <= -1 + 5

x <= 4

Or

x - 5 >= 1

x >= 1 + 5

x >= 6

In interval notation, the answer is:

[tex](-\infty,4\rbrack\cup\lbrack6,+\infty)[/tex]

The solution on the number line is:

5) 40,20,10,5, _,_,_a) Explain and Complete the sequence.B) write an explicit and recursive formula for the sequence

Answers

We have the sequence: 40, 20, 10, 5,...

Each term is half the previous term, so it is a geometrical sequence with common ratio r = 0.5.

We can not complete the sequence, as it becomes infinitely smaller and does not have a last term.

But we can write the three next terms to complete the blank spaces: 2.5, 1.25, 0.625.

We can start by writing the recursive formula. We know that each term is half the value of the previous term, so we wil have:

[tex]a_n=0.5\cdot a_{n-1}[/tex]

From this recursive formula, we can deduce the explicit formula (in terms of n) as:

[tex]\begin{gathered} a_1=40 \\ a_2=0.5\cdot40=20 \\ a_3=0.5\cdot20=0.5\cdot(0.5\cdot40)=0.5^2\cdot40=10 \\ a_4=0.5\cdot10=0.5\cdot(0.5^2\cdot40)=0.5^3\cdot40 \\ \Rightarrow a_n=40\cdot0.5^{n-1} \end{gathered}[/tex]

Answer:

a) Geometric sequence with r = 0.5.

The sequence first terms are: 40, 20, 10, 5, 2.5, 1.25, 0.625.

b) The recursive formula is a(n) = 0.5*a(n-1).

The explicit formula is a(n) = 40*0.5^(n-1).

The test results for 4 students are 96 83 78 and 83. If one more student's test score of 87 is added, what would increase?A. median B. meanC. modeD. range

Answers

Mean will increase because 87 is greater than 83 and 78, then the eman will be greater

0.4(2-) 0.2(9 + 7) A)-3 B - 1 C) 3 D) all real numbers

Answers

Let us solve the equation to arrange the steps

[tex]-3(4+3x)+5x=-16[/tex]

In the first step, we must multiply the bracket by -3 (distributive property)

[tex](-3)(4)_{}+(-3)(3x)=-12-9x[/tex]

Then the equation is

[tex]-12-9x+5x=-16[/tex]

Now add the like terms on the left side

[tex]\begin{gathered} -12+(-9x+5x)=-16 \\ -12x+(-4x)=-16 \\ -12-4x=-16 \end{gathered}[/tex]

Next step, add 12 to both sides

[tex]undefined[/tex]

The table shows the outcome of car accidents in a certain state for a recent year by whether or not the driver wore a seat belt. Find the probability of wearing a seat belt, given that the driver did not survive a car accident. Part 1: The probability as a decimal is _ (Round to 3 decimal places as needed.) Part 2: The probability as a fraction is _

Answers

Conditional probability is a measure of the probability of an event occurring, given that another event has already occurred.

The table shows the outcome of car accidents by whether or not the driver wearing a seat belt.

Let's call:

A = The event of the driver wearing a seat belt in a car accident.

B = The event of the driver dying in a car accident

The conditional probability is calculated as follows:

[tex]P(A|B)=\frac{P(A\cap B)}{P(B)}[/tex]

The conditional probability stated in the formula is that for the driver wearing a seat belt knowing he did not survive the car accident.

The numerator of the formula is the probability of both events occurring, i.e., the driver wore a seat belt and died. The denominator is the simple probability that the driver died in a car accident.

From the table, we can intersect the first column and the second row to find the number of outcomes where both events occurred. The probability of A ∩ B is:

[tex]P(A\cap B)=\frac{511}{583,470}[/tex]

The probability of B is:

[tex]P(B)=\frac{2217}{583,470}[/tex]

The required probability is:

[tex]P(A|B)=\frac{\frac{511}{583,470}}{\frac{2217}{583,470}}[/tex]

Simplifying the common denominators:

[tex]P(A|B)=\frac{511}{2217}=0.230[/tex]

Sarah Meeham blends coffee for Tasti-Delight. She needs to prepare 190 pounds of blended coffee beans selling for
$4.55 per pound. She plans to do this by blending together a high-quality bean costing $5.50 per pound and a cheaper
bean at $3.50 per pound. To the nearest pound, find how much high-quality coffee bean and how much cheaper coffee
bean she should blend.
She should blend lbs of high quality beans.
(Round to the nearest pound as needed.)

Answers

By solving an equation we can say that Sarah needs to blend a total of 96 pounds of coffee beans.

What are equations?A mathematical equation is a formula that uses the equals sign to represent the equality of two expressions. A mathematical statement that has an "equal to" symbol between two expressions with equal values is called an equation. As in 3x + 5 Equals 15, for instance. Equations come in a variety of forms, including linear, quadratic, cubic, and others. The point-slope form, standard form, and slope-intercept form are the three main types of linear equations.

So, the total pounds Sarah needs to blend:

Let the number of pounds by 'x'.

Now form an equation as:

5.50x + 3.50x = 190 × 4.55

Then, solve equation for 'x' as follows:

5.50x + 3.50x = 190 × 4.559x = 864.5x = 864.5/9x = 96 pounds


Therefore, by solving an equation we can say that Sarah needs to blend a total of 96 pounds of coffee beans.

Know more about equations here:

https://brainly.com/question/28937794

#SPJ13

Hi I need help with question 3 :) . Directions: For each real world situation, write and solve a system of equations . Give the solution as either an ordered pair or list what each variable is worth . Then explain what the solution means in terms of the situation

Answers

3.

We know that Hobby Land sells art supplies two different ways.

We can represent the situation with a system of equations

[tex]\begin{cases}x+y=139\ldots(1) \\ 4x+7y=781\ldots(2)\end{cases}[/tex]

Where x is the cost of one easel and y represents the cost of one paint set.

Now, we must solve the system of equations.

We can multiply equation (1) by -4

[tex]\begin{gathered} -4(x+y)=-4(139) \\ -4x-4y=-556\ldots(3) \end{gathered}[/tex]

Then, we can add (3) + (2)

[tex]\begin{gathered} -4x-4y=-556 \\ 4x+7y=781 \\ -------------- \\ 3y=225 \end{gathered}[/tex]

Now, we can solve the equation for y

[tex]\begin{gathered} 3y=225 \\ y=\frac{225}{3}=75 \end{gathered}[/tex]

Finally, to find x we can replace the value of y in the equation (1)

[tex]\begin{gathered} x+75=139 \\ x=139-75=64 \end{gathered}[/tex]

So, the cost of one easel is $64.

Solution as either an ordered pair:

- (64, 75).

Identify the composition that is represented by:r (90, O). T (-2, 4)A translation left 2, up 4 and then a reflection of 90°O A rotation of 90° and then a translation left 2, up 4.A reflection of 90° and then a translation left 2, up 4.O A translation of left 2, up 4 and then a rotation of 90°.

Answers

ANSWER:

A rotation of 90° and then a translation left 2, up 4.

STEP-BY-STEP EXPLANATION:

Since r (90, 0) is the first and means a 90 ° rotation and that T (-2, 4) is a translation of 2 units to the left (because it is negative) and 4 units up (because it is positive) , the answer is the option "A rotation of 90° and then a translation left 2, up 4."

Can someone do it for me please

Answers

Step-by-step explanation:

13.

a/7 + 5/7 = 2/7

a/7 = 2/7 - 5/7 = -3/7

a = -3

14.

6v - 5/8 = 7/8

6v = 7/8 + 5/8 = 12/8

v = 12/8 / 6 = 2/8 = 1/4

15.

j/6 - 9 = 5/6

j - 54 = 5

j = 5 + 54 = 59

16.

0.52y + 2.5 = 5.1

0.52y = 5.1 - 2.5 = 2.6

y = 2.6/0.52 = 5

17.

4n + 0.24 = 15.76

4n = 15.76 - 0.24 = 15.52

n = 15.52/4 = 3.88

18.

2.45 - 3.1t = 21.05

-3.1t = 21.05 - 2.45 = 18.6

t = 18.6/-3.1 = -6

simplify 3p x 5q x 2

Answers

30pq=3p×5q=15pq×2=30pq

What is the slope of (-8, -3) and (-7, -6)

Answers

To find the slope, we use the following formula.

[tex]m=\frac{y_2-y_1}{x_2-x_1}[/tex]

Let's replace the given points.

[tex]m=\frac{-6-(-3)}{-7-(-8)}=\frac{-6+3}{-7+8}=\frac{-3}{1}=-3[/tex]Hence, the slope is -3.

I just want to go to sleep but I need the answer to this question

Answers

The average rate of change of a function f(x) from x1 to x2 is given by:

[tex]\frac{f(x_2)-f(x_1)}{x_2-x_1}[/tex]

In this case we need the first three seconds so x1=0 and x2=3.

Calculate the values of the function at x=0 and x=3 to get:

f(0)=150 and f(3)=0.

Substitute these values into the formula for average rate of change:

[tex]\begin{gathered} \frac{f(3)-f(0)}{3-0} \\ =\frac{0-150}{3} \\ =\frac{-150}{3} \\ =-50 \end{gathered}[/tex]

Hence the avearage rate of change of the function for the first three seconds is -50.

Note that the negative sign shows that the function is decreasing in the time interval (first three seconds).

9b 9a) Use the slope formula to determine the rate of change eq y- and find the y-intercept "5" by substituting the x and y values into y=mx + b

Answers

A) We need to find the rate of change of the function first.

The rate of change or slope of the line is:

[tex]m=\frac{y_2-y_1}{x_2-x_1_{}}[/tex]

Where x and y are the coordinates of a point in line.

In order to calculate the slope we can take the poinst:

x1 = -6, y1 = 4

x2 = -2, y2= 1

Using the formula of above we find that the slope is:

[tex]m=\frac{1-4}{-2-(-6)}=-\frac{3}{4}[/tex]

Now, in order to find the value of y-intercept of the line we can use formula:

[tex]y=m\cdot x+b[/tex]

Which is the function of the line. From the formula of above we don't know the value of b (the y-intercept).

But we know that the formula must be valid for a point in the line. We can find the value of b replacing the coordinates of a point in the line, let's choose: x = -6 and y = 4, so:

[tex]4=\text{ m}\cdot(-6)+b[/tex]

Now we use the value of m of above:

[tex]4=(-\frac{3}{4})\cdot(-6)+b[/tex]

And from the last equation we can see that:

[tex]b=4-\frac{3}{4}\cdot6=4-\frac{9}{2}=\frac{8}{2}-\frac{9}{2}=-\frac{1}{2}[/tex]

So, the equation of the line is:

[tex]y\text{ = -}\frac{\text{3}}{4}\cdot x-\frac{1}{2}[/tex]

And the y-intercept is obtain replacing x = 0, so the y-intercept is: y = -1/2

b) From the stepts of above we already know an equation that represents the function! It is:

[tex]y\text{ = -}\frac{\text{3}}{4}\cdot x-\frac{1}{2}[/tex]

c) Now, we need to use the last equation to find y = n in the table. We know from the table that the value x for that value of y is x = 3, so we replace that value in the equation of the line:

[tex]y\text{ = -}\frac{\text{3}}{4}\cdot3-\frac{1}{2}=-\frac{9}{4}-\frac{1}{2}=-\frac{9}{4}-\frac{2}{4}=-\frac{11}{4}[/tex]

So the value of n is:

[tex]n\text{ = -}\frac{\text{11}}{4}[/tex]

Let p = x^2 + 6.Which equation is equivalent to (22 + 6)^2 – 21 = 4x^2 + 24 in terms of p?Choose 1 answer:А) p^2 + 4p - 21 = 0B) p^2 - 4p - 45 = 0C) p^2 - 4p - 21 = 0D) p^2 + 4p - 45 = 0

Answers

Given:

[tex](22+6)^2-21=4x^2+24[/tex][tex]\text{Let p = x}^2+6[/tex]

Let's solve the equation in terms of p:

[tex]undefined[/tex]

Other Questions
Select the polynomial functions for which (x+3) is a factor. Select all that apply. Someone pls quickly answer these. In Gibbons v. Ogden, the Supreme Court ruled that as a result of glycolysis, pyruvate oxidation and the citric acid cycle, only a small portion of the energy of glucose has been converted to atp. at this point, the majority of the usable energy is contained in: What causes body water to increase during pregnancy? GET BRAINLIEST What is called when totalitarian dictators produce movies, posters, and other forms of media to enhance their image? a sales invoice included the following information: merchandise price, $5,000; freight, $900; terms 1/10, n/eom, fob shipping point. assuming that a credit for merchandise returned of $700 is granted prior to payment and that the invoice is paid within the discount period, what is the amount of cash that should be received by the seller? The diagram shows a quadrilateral ABCD with each of its sides extended. AB = ADShow that ABCD is a kite. Give a reason for each stage of your working (The diagram is attached) Thanks, by neve124 Which of the following cognitive abilities declines during adulthood? crystallized intelligence wisdom fluid intelligence verbal abilitiestest review need help 3x - 7 = 3(x - 3) + 2 y+2=3(x4)y, plus, 2, equals, minus, 3, left parenthesis, x, minus, 4, right parenthesis Complete the missing value in the solution to the equation. is H less than 9?[tex]h \leqslant 9[/tex] This is matching:#1 If solving a problem with population growth compounding CONTINUOUSLY, which of the following formulas would you use?#2 If solving a problem with population growth compounding ANNUALLY, which of the following formulas would you use?#3 If solving a problem with population growth compounding QUARTERLY, which of the following formulas would you use?#4 If solving a problem with continuously compounding interest, which of the following formulas would you use?A: A(t)=P(1+rn)^ntB: A(t)=Pe^rtC: P(t)=P0(1+r)^tD: P(t)=P0^e^rt What type of orientation is being used if a document is wider than it is tall?A.MarginalB.IndentedC.LandscapeD.Portrait which of the following best describes XWO A. Angle bisectorB. MedianC. AltitudeD. Perpendicular bisector15W15Z there are methods that help measure application blank information by the degree to which the information differentiates between high- and low-performing individuals. this scoring methodology is called a(n G(x)=|x-2|How do u solve this? 1.what is the impact on development from malnutrition that begins after age 3? a.children will likely develop permanent and severe cognitive deficits. b.children will develop temporary physical deficits but not cognitive deficits. c.children will not likely experience permanent cognitive or behavioral deficits. d.children will develop temporary cognitive deficits but not physical deficits. a membrane protein in a plant root uses atp to transport nitrate ions from a fertilizer solution in the soil and concentrates them in the cell. this process is called: Assuming a fixed hourly pay rate, how much would an employee earn for working 4 hours based on this wage table? Hourly Pay Table Hours Worked 2 Money Earned $12.00 $24.00 $36.00 ? A. $36.00 B. $45.00 C. $64.00 D. $48.00